Quantcast
  • Register
PhysicsOverflow is a next-generation academic platform for physicists and astronomers, including a community peer review system and a postgraduate-level discussion forum analogous to MathOverflow.

Welcome to PhysicsOverflow! PhysicsOverflow is an open platform for community peer review and graduate-level Physics discussion.

Please help promote PhysicsOverflow ads elsewhere if you like it.

News

PO is now at the Physics Department of Bielefeld University!

New printer friendly PO pages!

Migration to Bielefeld University was successful!

Please vote for this year's PhysicsOverflow ads!

Please do help out in categorising submissions. Submit a paper to PhysicsOverflow!

... see more

Tools for paper authors

Submit paper
Claim Paper Authorship

Tools for SE users

Search User
Reclaim SE Account
Request Account Merger
Nativise imported posts
Claim post (deleted users)
Import SE post

Users whose questions have been imported from Physics Stack Exchange, Theoretical Physics Stack Exchange, or any other Stack Exchange site are kindly requested to reclaim their account and not to register as a new user.

Public \(\beta\) tools

Report a bug with a feature
Request a new functionality
404 page design
Send feedback

Attributions

(propose a free ad)

Site Statistics

205 submissions , 163 unreviewed
5,047 questions , 2,200 unanswered
5,345 answers , 22,709 comments
1,470 users with positive rep
816 active unimported users
More ...

  Are all infinite sums not divergent? In quantum field theory

+ 4 like - 0 dislike
1872 views

I am a physicist interested in physics. In particular this question is related to quantum field theory.

I recently came across a derivation of the infinite sum $1+1+1+1+..... $ that produced the result -1/2, aka zeta regularization (from Terry Tao's blog)

This was quite surprising to me as I had previously known an infinite sum of 1s to be divergent - from taking a math physics course by the guy that wrote "the book" on asymptotic methods.

(Indeed I've known the sum of all positive integers to be finite for quite some time as well as many other "divergent-looking" sums and I get the whole idea behind summation methods like Padé, Shanks, Euler, etc.)

Anyways this prompted me to wonder;

  1. Are ALL infinite sums not divergent?
  2. If not then how can one determine whether a sum os divergent or not?
  3. What was all this business in undergrad calculus about learning tests of convergence and all this business in complex analysis about series if weird things like $1+1+1+1+.....$ are actually convergent??

I'm still confused by all this stuff. And I haven't found an answer to these questions that "click"

Any help understand this topic would be kindly appreciated.

This post imported from StackExchange Mathematics at 2016-07-10 19:38 (UTC), posted by SE-user user122066
asked Jul 9, 2016 in Mathematics by user122066 (45 points) [ no revision ]
arxiv.org/abs/hep-ph/0510142

This post imported from StackExchange Mathematics at 2016-07-10 19:38 (UTC), posted by SE-user Count Iblis
You must distinguish between "the sum converges (or diverges)" and "we can assign a meaningful value to the sum via regularization". Those are two completely different things.

This post imported from StackExchange Mathematics at 2016-07-10 19:38 (UTC), posted by SE-user Asaf Karagila

2 Answers

+ 7 like - 0 dislike

No, many infinite sums converge. An infinite sum (or "series") $a_0 + a_1 + a_2 + \dots$ is defined to converge to a value $S$ if the limit $$ S = \lim_{n\rightarrow \infty} \sum_{i=1}^n a_i$$ exists. For example, if the value of $a_i$ falls off exponentially quickly or as a power law faster than $1/i$, then the series converges. The various convergence tests you learned in calculus can give you more precise criteria for convergence.

The infinite series $1 + 1 + 1 + \dots$ is not convergent - the above limit does not exist. However, it is regularizable - that is, you can play some tricks on it that "beat it into shape" well enough that you can assign some finite number to it. But this finite number is not actually the sum, which does not exist. Being regularizable is a much weaker criterion than being convergent.

So whenever you come across a divergent series in QFT and replace it with its regularized value, it's very important that you take into account that the two quantities aren't actually equal. Despite its being very important, there are approximately zero physicists who actually do it.

Edit: the OP asked a very good question in the comments that I though was worth addressing in my main answer: whether imposing different regulators on the same divergent series always yields the same result. If anyone has any thoughts, I've posed that question at http://math.stackexchange.com/questions/1854642/can-different-choices-of-regulator-assign-different-values-to-the-same-divergent. Also, I once asked a related question at How can dimensional regularization "analytically continue" from a discrete set? for which I never got a satisfactory answer.

This post imported from StackExchange Mathematics at 2016-07-10 19:38 (UTC), posted by SE-user tparker
answered Jul 9, 2016 by tparker (305 points) [ no revision ]
Thanks! This makes so much sense AND you didnt get really mad at me! I know many infinite sums converge. What i dont apparently understand is regularization is not the same as convergence. I thought regularization was a way to obtain convergence. Now are all infinite sums regularizable?

This post imported from StackExchange Mathematics at 2016-07-10 19:38 (UTC), posted by SE-user user122066
So what would the sum 2+4+8+16+32.....= -2 fall under? This is obtained without regulators but the sum is divergent (according to freshman calculus)

This post imported from StackExchange Mathematics at 2016-07-10 19:38 (UTC), posted by SE-user user122066
@user122066 Ah, I see your confusion now. No, regularization does not give convergence. Some people might say something like "we can use $\zeta$-regularization to show that the series converges to ..." but they're being sloppy with their language, the series still doesn't converge no matter what clever regularization you use. Regularization is a way to assign a finite value to a divergent series that is somewhat useful, but it doesn't actually sum the series.

This post imported from StackExchange Mathematics at 2016-07-10 19:38 (UTC), posted by SE-user tparker
@user122066 No, not all infinite sums are regularizable. There is no way to assign a finite value to the harmonic series $1 + 1/2 + 1/3 + 1/4 + \dots$ even if you try to use $\zeta$-regularization, because you end up hitting a pole in the Riemann zeta function.

This post imported from StackExchange Mathematics at 2016-07-10 19:38 (UTC), posted by SE-user tparker
@user122066 The series $2 + 4 + 8 + 16 + \dots$ does not converge, as can be shown by any convergence test. (The terms have to decrease to zero for convergence to even by possible.) If you look at the formula you used to sum the series, you'll note that it only actually applies if the base of the exponent in the geometric sequence is less than 1, which isn't the case here. So strictly speaking, your formula does not apply. In order to get the value -2, you unknowingly used analytic continuation to assign a finite value to the divergent series.

This post imported from StackExchange Mathematics at 2016-07-10 19:38 (UTC), posted by SE-user tparker
@user122066 This is closely analogous to using a regulator, you just didn't know you were doing it.

This post imported from StackExchange Mathematics at 2016-07-10 19:38 (UTC), posted by SE-user tparker
i can analytically continue or i can do: S=2+4+8+....= 2 + 2×S => S = -2. But yeah. You get this sum by simply using linearity and being able to pull out a 2.

This post imported from StackExchange Mathematics at 2016-07-10 19:38 (UTC), posted by SE-user user122066
This stuff youve said here makes so much sense it hurts!

This post imported from StackExchange Mathematics at 2016-07-10 19:38 (UTC), posted by SE-user user122066
In the same way ramanujan finds 1+2+3+4+.... but i get your point. They arent convergent....they just have values. Whoch begs the question: they can just be ANY values.....what causes them to have UNIQUE values?

This post imported from StackExchange Mathematics at 2016-07-10 19:38 (UTC), posted by SE-user user122066
@user122066 Yeah, that whole "formally replacing the series with S and then algebraically manipulating S" trick is actually just heuristic because the series doesn't actually converge so S isn't well-defined. And it can sometimes get you into trouble for more complicated series. Analytic continuation is the more rigorous way to do it.

This post imported from StackExchange Mathematics at 2016-07-10 19:38 (UTC), posted by SE-user tparker
@user122066 Regarding whether the sum is unique, in the sense that every choice of regulator gives you the same answer: that's a great question - I've wondered the exact same thing myself and I don't know the answer.

This post imported from StackExchange Mathematics at 2016-07-10 19:38 (UTC), posted by SE-user tparker
@user122066 I edited my main answer with some links to related questions.

This post imported from StackExchange Mathematics at 2016-07-10 19:38 (UTC), posted by SE-user tparker
+ 1 like - 0 dislike

Read more carefully the first part of Terry Tao's post. He replaces the regular partial sums with a smoothed sum

$$\sum_{n=1}^N n^s \to \sum_{n=1}^\infty \eta(n/N) n^s$$

where $\eta$ is a cutoff function. The result he finds for the smoothed sum of $1+1+1+\dots$ (case $s=0$) is

$$\sum_{n=1}^\infty \eta(n/N) = - \frac 1 2 + C_{\eta,0} N + O(1/N)$$

where

$$C_{\eta,0} =\int_0^\infty \eta(x) dx$$

On the right side there is an asymptotic expansion of the smoothed sum. As you can see, there is indeed a constant term $-1/2$, but the following term is divergent in the limit $N \to \infty$. So it is misleading to state that

$$1+1+1+\dots = -1/2$$

because $-1/2$ is just the constant term of an asymptotic expansion which is divergent in the limit $N\to \infty$.

What is usually done in QFT (see Luboš Motl's answer here for example) is to cancel the leading divergence by means of a local counterterm. Basically, a "trick" is used to ged rid of the divergence and be able to write $1+1+1+\dots=-1/2$.

This post imported from StackExchange Mathematics at 2016-07-10 19:38 (UTC), posted by SE-user valerio92
answered Jul 10, 2016 by valerio92 (10 points) [ no revision ]

Your answer

Please use answers only to (at least partly) answer questions. To comment, discuss, or ask for clarification, leave a comment instead.
To mask links under text, please type your text, highlight it, and click the "link" button. You can then enter your link URL.
Please consult the FAQ for as to how to format your post.
This is the answer box; if you want to write a comment instead, please use the 'add comment' button.
Live preview (may slow down editor)   Preview
Your name to display (optional):
Privacy: Your email address will only be used for sending these notifications.
Anti-spam verification:
If you are a human please identify the position of the character covered by the symbol $\varnothing$ in the following word:
p$\hbar$y$\varnothing$icsOverflow
Then drag the red bullet below over the corresponding character of our banner. When you drop it there, the bullet changes to green (on slow internet connections after a few seconds).
Please complete the anti-spam verification




user contributions licensed under cc by-sa 3.0 with attribution required

Your rights
...